Foro de preguntas y respuestas de Matemáticas

logo beUnicoos
Los foros de unicoos se han unificado en nuestra nueva plataforma beUnicoos. Para dejar nuevas preguntas deberás hacerlo allí, donde además podrás encontrar nuevas asignaturas y herramientas para ayudarte más con tus estudios.

  • icon

    Diego Schwindt
    el 27/11/17

    Me piden racionalizar. Muchas gracias!!!!



    replythumb_up0 voto/sflag
    icon

    Antonius Benedictus
    el 27/11/17


    thumb_up1 voto/sflag
  • Usuario eliminado
    el 27/11/17
    flag

    Unicoos por favor me podrian ayudar con la demostracion del teorema de weierstrass. Muchas gracias


    replythumb_up0 voto/sflag
    icon

    Antonius Benedictus
    el 27/11/17

     

    ¡Hola! Nos encantaría ayudarte, pero no solemos responder dudas universitarias que no tengan que ver específicamente con los vídeos que David Calle ha grabado como excepción. O de otras asignaturas que no sean Matemáticas, Física y Química. Lo sentimos de corazón… Esperamos que  lo entiendas.

    Ojalá algún unicoo universitario se anime a ayudarte (de hecho, lo ideal es que todos los universitarios intentarais ayudaros los unos a los otros).

    thumb_up1 voto/sflag
  • Usuario eliminado
    el 27/11/17

    Hola Unicoos he resuelto estos ejercicios y me ha salido:

    7) 8 meses

    8) √5/3


    replythumb_up0 voto/sflag
    icon

    Antonio Silvio Palmitano
    el 27/11/17

    7)

    Puedes considerar que la función del tiempo: f(t) = "peso del niño, expresado en libras" es una función continua, ya que los cambios de peso segundo a segundo son imperceptibles.

    Luego, tienes el intervalo cerrado temporal: [0,12], expresado en meses

    Luego, observa que tienes:

    f(0) = 8,

    f(12) = 16.

    Luego, tienes que el peso en estudio está comprendido entre los valores que toma la función en los extremos del intervalo:

    ≤ 11 ≤ 16,

    y luego, como se cumplen las hipótesis (la función es continua y el intervalo es cerrado), de acuerdo con el Teorema del Valor Intermedio, tienes que existe c perteneciente al intervalo, tal que:

    f(c) = 11.

    Observa que en el enunciado te piden demostrar que existe un instante c para el cual el peso del niño es 11 libras, pero no te piden calcularlo.

    Espero haberte ayudado.


    thumb_up1 voto/sflag
    icon

    Antonio Silvio Palmitano
    el 27/11/17

    8)

    Tienes la ecuación:

    x3 - 3x + 1 = 0, que es una ecuación polinómica cúbica,

    y debes demostrar que tiene una solución real en el intervalo cerrado: [0,1].

    Luego, observa que en el primer miembro de la ecuación tienes la expresión de la función:

    f(x) = x3 - 3x + 1, que es una función continua,

    y en los extremos del intervalo cerrado toma los valores:

    f(0) = 1 > 0,

    f(1) = -1 < 0;

    luego, observa que 0 es un valor intermedio entre los valores que toma la función en el intervalo cerrado,

    por lo que aplicas el Teorema de Bolzano, y tienes que existe c perteneciente al intervalo, tal que:

    f(c) = 0, sustituyes la expresión de la función evaluada, y queda:

    x3 - 3c + 1 = 0,

    por lo que tienes que la ecuación tiene al menos una solución real c en el intervalo [0,1].

    Espero haberte ayudado.

    thumb_up1 voto/sflag
  • icon

    Gabriela Peraza
    el 27/11/17

    Hola, ¿me pueden ayudar con este ejercicio?, es de sólidos de revolución por el método de discos.

    Un tanque de aceite de forma esférica  tiene 20m de diámetro. ¿Cuánto aceite contiene si la profundidad del mismo es de 8m?


    replythumb_up0 voto/sflag
    icon

    Antonius Benedictus
    el 27/11/17


    http://julian48.webcindario.com/mates/casquetesf/Casquete%20esf%C3%A9rico.htm

    thumb_up1 voto/sflag
  • icon

    MÓNICA VIVIANA chandía
    el 27/11/17

    Hola. Alguien me puede decir si hice bien el ejercicio? y me da dos resultados distintos pero ahora no sé cual es el correcto (o ambos están mal)

    replythumb_up0 voto/sflag
    icon

    Livaldo Ocanto
    el 27/11/17

    La priemra es la correcta, da 1.

    thumb_up1 voto/sflag
    icon

    Ángel
    el 27/11/17


    thumb_up2 voto/sflag
    icon

    Ángel
    el 27/11/17

    Si el enunciado te pide las asíntotas horizontales de una función, hay dos: una en x=1 cuando x tiende a infinito y otra en x= 7/3 cuando x tiende a menos infinito.

    Manda el enunciado original para tener certeza.


    thumb_up2 voto/sflag
    icon

    MÓNICA VIVIANA chandía
    el 27/11/17


    thumb_up0 voto/sflag
    icon

    MÓNICA VIVIANA chandía
    el 27/11/17

    Ahí mandé en imágen el ejercicio.

    Si el resultado es 1, entonces hay una asintota horizontal. Estoy en lo correcto?

    thumb_up0 voto/sflag
    icon

    MÓNICA VIVIANA chandía
    el 1/12/17

    hola. Nuevamente tengo dificultad para resolver límites. Me podrán ayudar?

    thumb_up0 voto/sflag
  • icon

    Livaldo Ocanto
    el 27/11/17

    En problemas de nuevo, che no puedo romper la pinche indeterminacion :(((( tiene que ser sin aplicar L'hopital

    replythumb_up0 voto/sflag
    icon

    Antonius Benedictus
    el 27/11/17


    thumb_up0 voto/sflag
    icon

    Livaldo Ocanto
    el 27/11/17

    Me di cuenta que hay un error, es es 1 - la tangente elevado a la 3.

    thumb_up0 voto/sflag
  • icon

    Guido Ferrari
    el 27/11/17

    Hola, alguien me ayudaría pls? Me lo toman mañana esto 


    replythumb_up0 voto/sflag
    icon

    Neofito 007
    el 27/11/17

    Usted está llevando métodos numéricos ?  de lo contrario con ninguna propiedad de logaritmo va poder resolverlo y hallar x .

    thumb_up0 voto/sflag
    icon

    Guido Ferrari
    el 27/11/17

    Hola, aparentemente tengo que utilizar una formul para resolver la ecuacion pero la desconozco, falte a clase porque estaba enfermo. :/

    thumb_up0 voto/sflag
  • icon

    Diego Salazar
    el 27/11/17

    ayuda

    replythumb_up0 voto/sflag
    icon

    Livaldo Ocanto
    el 27/11/17

    Use completacion de cuadrado para factorizar el polinomio dentro de la raiz

    thumb_up0 voto/sflag
  • icon

    Rubén
    el 27/11/17

    Hola, ¿me pueden ayudar con este ejercicio?


    replythumb_up0 voto/sflag
    icon

    Antonius Benedictus
    el 27/11/17


    thumb_up1 voto/sflag